User avatar
 
LSAT-Chang
Thanks Received: 38
Atticus Finch
Atticus Finch
 
Posts: 479
Joined: June 03rd, 2011
 
 
trophy
Most Thankful
trophy
First Responder
 

Q20 - Productivity is average output

by LSAT-Chang Mon Aug 08, 2011 1:22 pm

Could someone explain to me why (B) is the correct answer?

Here is my thought process:

I first picked (E) when solving this whole section timed, and when I went to solve it the second time very carefully (untimed and before looking at answer sheet)), I picked (A).

The reason I eliminated (E) was because the author does not "assume that high-tech equipment cannot contribute to increases in productivity" but rather just that high productivity is not dependent on having high-tech equipment. So the author could still believe that it contributes to it, but just isn't dependent on it.

I chose (A) the second time -- although I still wasn't sure what (A) was even talking about (I just thought it had something to do with sufficient/necessary since the argument had something about the word "depend" which is a necessary condition.. and none of the other answers seemed attractive to me).

The argument core to me looks like this:

Because high productivity cannot be achieved without adequate training of workers, high productivity doesn't depend on having high-tech equipment.

The "flaw" that popped out at me immediately was that the author was ignoring the idea that high-tech equipment could still be another necessary factor for high productivity. It seemed like the author was just assuming that adequate training of workers and having high-tech equipment couldn't both contribute to high productivity. So I was looking for an answer choice something like, "ignores the possibility that having high-tech equipment is another necessary factor for high productivity".

What I don't understand is how (B) is a flaw of the argument. Why does the author HAVE to take into consideration the possibility that having high-tech equipment is required for adequate training of workers? If we are linking the two random variables "adequate training of workers" and "high-tech equipment" because it is a mismatch or something, why do we have to do this? And if this is the case, for flaw questions, are we looking for an answer that addresses both terms (one from premise and the other from conclusion)?
 
timmydoeslsat
Thanks Received: 887
Atticus Finch
Atticus Finch
 
Posts: 1136
Joined: June 20th, 2011
 
This post thanked 3 times.
 
trophy
Most Thanked
trophy
First Responder
 

Re: Q20 - Productivity is average output

by timmydoeslsat Mon Aug 08, 2011 8:35 pm

My favorite question that you have posted. This is because you have really fought on this one. I can tell.

This stimulus is an argument and we want to find the flaw.

Let us get to the core.

The core can be seen as this:

High productivity can be reached ---> Adequate training of workers

THEREFORE ---->

High productivity does not depend on having high tech equipment


Huge issue with the reasoning here.

You were right on the money with your thought process! A correct answer could have been what you said! It could be the case that high tech equipment is another necessary condition of high productivity being reached.

Who knows how many necessary conditions there are with reaching high productivity.

You can think of it like this, of course,

Living cannot be achieved for a human without oxygen.

Living for a human ---> Oxygen

Notice that we also have tons of other things that are necessary for a human to live! I would say blood, heart beat, etc...

The answer choice on this particular problem is probably why this is a #20 instead of a #2.

Let us talk about why answer choice B is correct in this problem.

What do we know?

High productivity can be reached ---> Adequate training of workers

We know that we MUST have adequate training of workers for us to have high productivity being reached, don't we?

Great. What do we know when we have a case of adequate training? NOTHING! It is a necessary condition in this case and we cannot conclude anything about it! We can conclude something if we do not have a case of adequate training. We would know that we would then, by necessity, not have a case of high productivity being reached.

Nonetheless, we KNOW THAT WE WILL BE HAVING a case a of adequate training if we have high productivity being reached. We just cannot conclude what necessary conditions adequate training could bring!

Couldn't it be possible that adequate training requires high tech equipment? It may also require horse shoes and tennis shoes, I do not know. But I do know that this argument cannot conclude ~ high tech equipment.

This is because adequate training may indeed require it!

Answer choices:

A) Common answer choice when dealing with these types of issues. However, notice that what is at issue here is the conclusion of ~high tech equipment and how the arguer reached that conclusion. This answer choice is talking about a necessary condition of high prod. being reached, which is adequate training.

Adequate training is never even mentioned again in this argument, so there is no way it is being confused for a sufficient condition. The arguer never talks about "if adequate training..."

B) High productivity may in fact depend on high tech equipment. We know high productivity requires adequate training. What does adequate training require? Could be high tech equipment!

C) Not a flaw in this argument. Who cares about desirability in this argument.

D) Does it assume the reversal of the stated conditional relationship? Absolutely not! Does it assume:

Adequate training ---> High productivity being reached

No! Plus, I am assuming that adequate training = educating workers. I think that is not a far leap to make, so I did without hesitation.

E) Does not assume that. The argument just said that high productivity does not depend on it.
User avatar
 
LSAT-Chang
Thanks Received: 38
Atticus Finch
Atticus Finch
 
Posts: 479
Joined: June 03rd, 2011
 
 
trophy
Most Thankful
trophy
First Responder
 

Re: Q20 - Productivity is average output per worker per unit

by LSAT-Chang Mon Aug 08, 2011 9:04 pm

timmydoeslsat Wrote:
Couldn't it be possible that adequate training requires high tech equipment? It may also require horse shoes and tennis shoes, I do not know. But I do know that this argument cannot conclude ~ high tech equipment.

This is because adequate training may indeed require it!


Timmy, your explanations are like magic -- not even exaggerating a single word! I swear! You just taught me a whole new "flexibility" key right here! You are SO damn right! I didn't even "think" about what adequate training requires! And because my mind was solely focused on the high tech equipment and high productivity, answer choice (B) sounded not even close since to me, it sounded like they just put together two variables that don't have anything to do with each other! So if we had (B), then we would have:

high productivity --> adequate training of works --> high-tech equipment

So automatically we would know that high productivity DOES in fact depend on high-tech equipment! So that is the flaw!

To be honest with you, this was a very tough problem for me and I honestly am not confident that I will be able to solve another similar problem like this correctly the next time -- but I will definitely try to be more flexible with my thinking. I feel like if I spot a flaw, then I only look for THAT flaw, and if I don't see it, then I start freaking out and have to go back to the core and that takes a whole lot of time -- when I could have just thought a little bit deeper! :shock:
 
Sully
Thanks Received: 0
Forum Guests
 
Posts: 2
Joined: March 02nd, 2013
 
 
 

Re: Q20 - Productivity is average output

by Sully Tue Jul 16, 2013 12:04 am

Hi guys,
Do you think answer choice A would be correct if we substituted "High productivity" wherever it says "high-tech equipment" Here's how I see it playing out:


Premise:
HP -----> AT

But if instead, we assume that AT is sufficient, rather than necessary:

AT ----> HP

The assumption leads to the conclusion that high-tech equipment is not necessary for high productivity.
 
foralexpark
Thanks Received: 2
Vinny Gambini
Vinny Gambini
 
Posts: 24
Joined: June 08th, 2013
 
 
trophy
First Responder
 

Re: Q20 - Productivity is average output

by foralexpark Sat Sep 21, 2013 5:10 pm

Sully Wrote:Hi guys,
Do you think answer choice A would be correct if we substituted "High productivity" wherever it says "high-tech equipment" Here's how I see it playing out:


Premise:
HP -----> AT

But if instead, we assume that AT is sufficient, rather than necessary:

AT ----> HP

The assumption leads to the conclusion that high-tech equipment is not necessary for high productivity.



Pretty clever!

I think it works, because by assuming the new relationship, the high-tech equipment would not be needed, so conclusion is stronger
User avatar
 
WaltGrace1983
Thanks Received: 207
Atticus Finch
Atticus Finch
 
Posts: 837
Joined: March 30th, 2013
 
 
trophy
Most Thanked
trophy
Most Thankful
trophy
First Responder
 

Re: Q20 - Productivity is average output

by WaltGrace1983 Mon Apr 14, 2014 5:02 pm

foralexpark Wrote:
Sully Wrote:Hi guys,
Do you think answer choice A would be correct if we substituted "High productivity" wherever it says "high-tech equipment" Here's how I see it playing out:


Premise:
HP -----> AT

But if instead, we assume that AT is sufficient, rather than necessary:

AT ----> HP

The assumption leads to the conclusion that high-tech equipment is not necessary for high productivity.



Pretty clever!

I think it works, because by assuming the new relationship, the high-tech equipment would not be needed, so conclusion is stronger


I think, however, that this would still ignore the most pivotal flaw in this argument. This flaw is, "Where did High-tech equipment come from?!" This idea just comes all the way from left field.
 
SJK493
Thanks Received: 1
Jackie Chiles
Jackie Chiles
 
Posts: 31
Joined: May 14th, 2018
 
 
 

Re: Q20 - Productivity is average output

by SJK493 Wed Aug 15, 2018 11:57 pm

Is answer (E) incorrect because it states presumes without giving justification that high-tech equipment cannot contribute to increases in productivity? It does not assume this, because this is explicitly stated in the conclusion, high productivity does not depend on having high-tech equipment. So this is eliminated because an assumption is an unstated premise.

Furthermore, would answer (E) be more acceptable if were ignores the possibility that high-tech equipment can contribute to increases in productivity? The term used in the conclusion 'depend on' does not necessarily mean that it is a necessary condition (conditional reasoning) which opens the possibility that may be causal in nature. Any thoughts on this?
 
RanW386
Thanks Received: 0
Vinny Gambini
Vinny Gambini
 
Posts: 4
Joined: May 03rd, 2019
 
 
 

Re: Q20 - Productivity is average output

by RanW386 Sat May 25, 2019 9:08 am

SJK493 Wrote:Is answer (E) incorrect because it states presumes without giving justification that high-tech equipment cannot contribute to increases in productivity? It does not assume this, because this is explicitly stated in the conclusion, high productivity does not depend on having high-tech equipment. So this is eliminated because an assumption is an unstated premise.

Furthermore, would answer (E) be more acceptable if were ignores the possibility that high-tech equipment can contribute to increases in productivity? The term used in the conclusion 'depend on' does not necessarily mean that it is a necessary condition (conditional reasoning) which opens the possibility that may be causal in nature. Any thoughts on this?


I agree that it does not assume this, but I think the main issue about E is that "A cannot contribute to B" is not the same as "B does not depend on A", reverse logic here
 
YufeiR103
Thanks Received: 0
Vinny Gambini
Vinny Gambini
 
Posts: 13
Joined: November 01st, 2022
 
 
 

Re: Q20 - Productivity is average output

by YufeiR103 Sun Dec 18, 2022 3:17 am

WaltGrace1983 Wrote:
foralexpark Wrote:
Sully Wrote:Hi guys,
Do you think answer choice A would be correct if we substituted "High productivity" wherever it says "high-tech equipment" Here's how I see it playing out:


Premise:
HP -----> AT

But if instead, we assume that AT is sufficient, rather than necessary:

AT ----> HP

The assumption leads to the conclusion that high-tech equipment is not necessary for high productivity.



Pretty clever!

I think it works, because by assuming the new relationship, the high-tech equipment would not be needed, so conclusion is stronger


I think, however, that this would still ignore the most pivotal flaw in this argument. This flaw is, "Where did High-tech equipment come from?!" This idea just comes all the way from left field.


At first, I think it would be right if changing "high-tech equipment" to "high productivity", however, I found it would not fit it in the case of B, which is a possible case.

Assume the case of B:
HE (High-tech equipment)----->AT------->HP,

Then according to A, the flaw is that some condition for HP, which is necessary, is thought to be sufficient.
Let's go back to the conclusion in the statement, which is HE is not necessary. And the conclusion's assumption is that AT itself is sufficient. So far everything is great, cause AT is the necessary condition though to be sufficient.
However, as B says, HE is required by AT, and that is if AT exists, HE also exists; and if HE doesn't exist, AT neither. So we need to reconsider the conclusion in the statement. It says HP can exist without HE, and that is also without AT (cause no HE, no AT), then the conclusion apparently didn't take AT as sufficient condition, since it assumes there is no AT , as a result, A doesn't work even with the substitution.